LSAT and Law School Admissions Forum

Get expert LSAT preparation and law school admissions advice from PowerScore Test Preparation.

 Administrator
PowerScore Staff
  • PowerScore Staff
  • Posts: 8916
  • Joined: Feb 02, 2011
|
#25836
Complete Question Explanation

Must Be True—CE. The correct answer choice is (A)

The stimulus contains a fact set discussing what has been done to protect wild animal displays in a natural history museum. The displays were created by drying and mounting animal skins. Some of the older displays are showing damage, with the animals’ skins having started to deteriorate because of low humidity and the heat of the tungsten lamps used for the display lights.

This fact about the deterioration of the displays creates a causal relationship. There are two causes of the deterioration: low humidity and heat (given off by the tungsten lamps). Next, we are told that the newer displays are not lit by tungsten lamps, but rather by compact fluorescent lamps designed for use in museums. These lamps produce less heat than the tungsten lamps.

The question stem tells you that this is a Must Be True question. For this type of question, an effective prephrase technique is to consider whether any of the information is discussed twice. If so, this repetition can create a bridge connecting the two facts, permitting an inference.

In this case, the idea of heat was mentioned in more than one fact. We know that heat from the tungsten lamps is a cause of deterioration in the older displays, and we know that the compact fluorescent lamps produce less heat than the tungsten lamps. So, your prephrase is that a display lit with a compact fluorescent lamp will suffer less deterioration from heat than a display lit with a tungsten lamp.

Answer choice (A): This is the correct answer choice. Because the heat from the tungsten lamps was contributing to the deterioration of some of the older displays, we can infer that when the excess heat from those lamps is removed, the rate of deterioration will slow.

Answer choice (B): This choice is incorrect because we do not know the precise difference in heat between the tungsten and fluorescent lamps, nor do we know whether there is some negative synergistic effect from using many fluorescent lamps to light a single display.

Answer choice (C): The stimulus does not provide any information regarding the number of tungsten lamps versus the number of fluorescent lamps used.

Answer choice (D): Since the stimulus dealt only with the lamps and said nothing about the source or remediation of the humidity affecting the displays, there is no support in the stimulus for this statement.

Answer choice (E): As in answer choice (D), the stimulus did not provide sufficient information regarding the humidity in the museum to support this inference. All you know from the stimulus is that humidity is one of the causes of the deterioration.
 DrewKing
  • Posts: 6
  • Joined: Aug 09, 2016
|
#28639
I understand how "A" is supports. But....it seems to obvious. I question answer choice B. The reason i feel it supports very well is because of the two words used in the answer choice, "Many" (large number) and "Few" (small number). The premise in the stimulus states the fluorescent are obviously better, they produce as much light as the tungsten, but Less heat!! (meaning they still produce heat) So..this is where wording i thought was imperative...In B, many compact fluorescent lamps vs. few tungsten lamps. Subsequently, equal light is produced as per the stimulus, but "many" compact fluorescent lamps on display will obviously give off more heat than "few" tungsten lamps on display. Is this an error?? Please help me in understanding this logic of thinking, thank you.
 Adam Tyson
PowerScore Staff
  • PowerScore Staff
  • Posts: 5153
  • Joined: Apr 14, 2011
|
#28779
Hey Drew, thanks for asking.

Sometimes, obvious answer are right answers. Maybe they are obvious because you have that good a grasp of the stimulus and stem, or maybe they are obvious just because they are easy questions (this test does have a few of those, usually found in the first 10 questions of any given LR section). Don't reject an answer because it's too obvious - reject it because it's not the best answer choice, and only because of that.

Here, answer B requires too much help and/or outside info to prove true. How much heat does a compact fluorescent put out compared to a tungsten? What if tungsten lamps put out a million units of heat (I skipped that year in science - what do you call a unit of heat? A therm? A hamp? A warm-a-mabob?) and a compact fluorescent puts out 1/10th of a unit? Or none at all? That's what's wrong with B - without knowing the relative warm-a-mabob output of each type, I can't calculate the numbers to support it. Besides, who said that any displays were lit by many lanps?

Here, obvious is good. Don't outsmart yourself by talking yourself out of a perfectly good answer!
 TheKingLives
|
#75049
Hi there, is D incorrect because the stimulus doesn't mention that the new displays are affected by low humidity? I ultimately chose A but was really split between A and D because I thought the low humidity affected the entire museum, and thus both the old and new displays.
 Paul Marsh
PowerScore Staff
  • PowerScore Staff
  • Posts: 290
  • Joined: Oct 15, 2019
|
#75184
Hi TheKingLives!

You said that "I thought the low humidity affected the entire museum, and thus both the old and new displays." I agree with you; the passage give us no reason to think that the new displays aren't also affected by low humidity. But Answer Choice (D) says the opposite of this - it says definitively that the new displays are not affected by low humidity. I'm wondering if maybe you missed the "not" in Answer Choice (D)?

Hope that helps! Feel free to follow up if you still have a question.
User avatar
 PresidentLSAT
  • Posts: 87
  • Joined: Apr 19, 2021
|
#102169
Help pls lol,

Is D wrong because deterioration isn't limited to the lighting source?
User avatar
 Jeff Wren
PowerScore Staff
  • PowerScore Staff
  • Posts: 389
  • Joined: Oct 19, 2022
|
#102177
Hi PresidentLSAT,

To be precise, Answer D is wrong because we don't know anything about how humidity relates to the new lamps based on the information provided in the stimulus.

All we know about the new lamps from the stimulus is that they give off as much light but less heat than the old tungsten lamps. Since heat from the lamps is one of the causes of the deterioration, replacing the old lamps with the newer lamps should reduce the rate of deterioration, which is what Answer A correctly states.

For Must Be True questions (including the Most Strongly Supported variation), the correct answer must be directly supported by the facts in the stimulus. Any answer that contains information that is unknown based on the stimulus (such as Answer D), is wrong.

Get the most out of your LSAT Prep Plus subscription.

Analyze and track your performance with our Testing and Analytics Package.